diff --git a/src/q4/circmes-ELEC1370/exam/2022/2022.mk b/src/q4/circmes-ELEC1370/exam/2022/2022.mk new file mode 100644 index 000000000..3f2bc1c33 --- /dev/null +++ b/src/q4/circmes-ELEC1370/exam/2022/2022.mk @@ -0,0 +1,2 @@ +YEAR=2022 +include ../../../exam.mk diff --git a/src/q4/circmes-ELEC1370/exam/2022/Juin/Juin.mk b/src/q4/circmes-ELEC1370/exam/2022/Juin/Juin.mk new file mode 100644 index 000000000..35bb47f16 --- /dev/null +++ b/src/q4/circmes-ELEC1370/exam/2022/Juin/Juin.mk @@ -0,0 +1,2 @@ +MONTH=Juin +include ../../2022.mk diff --git a/src/q4/circmes-ELEC1370/exam/2022/Juin/Majeure/Makefile b/src/q4/circmes-ELEC1370/exam/2022/Juin/Majeure/Makefile new file mode 100644 index 000000000..69a28ee8b --- /dev/null +++ b/src/q4/circmes-ELEC1370/exam/2022/Juin/Majeure/Makefile @@ -0,0 +1,2 @@ +MINMAJ=Majeure +include ../Juin.mk diff --git a/src/q4/circmes-ELEC1370/exam/2022/Juin/Majeure/circmes-ELEC1370-exam-2022-Juin-Majeure.tex b/src/q4/circmes-ELEC1370/exam/2022/Juin/Majeure/circmes-ELEC1370-exam-2022-Juin-Majeure.tex new file mode 100644 index 000000000..f55e34be2 --- /dev/null +++ b/src/q4/circmes-ELEC1370/exam/2022/Juin/Majeure/circmes-ELEC1370-exam-2022-Juin-Majeure.tex @@ -0,0 +1,106 @@ +\documentclass[fr]{../../../../../../eplexam} + +\hypertitle{circmes}{4}{ELEC}{1370}{2022}{Juin}{Majeure} +{Quentin De Laet} +{Christophe Craeye, Bruno Dehez and Claude Oestges} + + +\usepackage[oldvoltagedirection]{circuitikz} +\usepackage{amsmath} + +\begin{document} + +\section{Question Oestges : Bode et quadripôles} + +On considère le montage suivant : + +\begin{center} +\begin{circuitikz} + \draw (0,0) node[above]{$v_i$} to[short, o-*] ++(1,0) coordinate(SPL1) -- ++(0,-2) to[R=$R_H$] ++(2,0) to[C=$C_H$] ++(2,0) + node[op amp, anchor=-](OAH){} to[short,*-] ++(0,1.25) coordinate(OAHm) to[R=$R_H$] (OAHm -| OAH.out) to[short,-*] (OAH.out) + to[R=$R_i$] ++(2,0) coordinate(EndH) + (OAH.+) node[ground]{} + (SPL1) -- ++(0,1.5) to[R=$R_L$] ++(4,0) node[op amp, anchor=-](OAL){} to[short,*-] ++(0,1.25) coordinate(OALm) + to[R=$R_L$, *-*] (OALm -| OAL.out) + (OALm) -- ++(0,1.25) coordinate(OALm2) to[C=$C_L$] (OALm2 -| OAL.out) -- (OAL.out) to[R=$R_i$, *-] ++(2,0) + (OAL.+) node[ground]{} + (EndH) -- ++(0,2) coordinate(OAf_in) -- (OAf_in |- OAL.out) + (OAf_in) to[short, *-*] ++(1,0) node[op amp, anchor=-](OAf){} -- ++(0,1.25) coordinate(OAfm) to[R=$R_f$] (OAfm -| OAf.out) -- (OAf.out) + to[short,*-o] ++(1,0) node[above]{$v_o$} + (OAf.+) node[ground]{} + ; +\end{circuitikz} +\end{center} + +On demande : +\begin{enumerate} + \item le gain $g_f(\omega) = \frac{v_o}{v_i}$ du montage + \item le diagramme de Bode de ce montage (en dB et en phase) avec les valeurs suivantes (unités omises) : + \begin{itemize} + \item $R_f = 2R_i = 2000$ + \item $R_L C_L = 10^{-5}$ + \item $R_H C_H = 3\cdot10^{-6}$ + \end{itemize} + \item l'utilité de ce montage + \item les valeurs $Z_{in}$ et $Z_{out}$ du quadripôle lorsqu'une charge de valeur $Z_L$ est placée à sa sortie, + sachant que la source de tension connectée en entrée possède une résistance interne de valeur $R_S$. +\end{enumerate} + +\nosolution + +\newpage +\section{Question Dehez : circuits magnétiques couplés et puissance} + +Soit le montage suivant, fonctionnant à une fréquence de $50Hz$ : + +\begin{center} +\begin{circuitikz}[american] + \draw (0,0) to[V, l=$24V\angle 0$] ++(0,2) -- node[flowarrow]{$\Bar{I_a}$} ++(1,0) -- ++(2,0) to[R=$1\Omega$] ++(2,0) coordinate(SPLIT) + to[american inductor=$j2\Omega$] node[below](a){$\bullet$} ++(2,0) to[C=$-j2\Omega$] ++(2,0) node[below](b){$\bullet$} + to[american inductor=$j2\Omega$] ++(2,0) to[R=$1\Omega$] ++(0,-2) -- (0,0); + \draw (SPLIT) to[C=$-j1\Omega$,*-*] ++(0,-2); + \draw (a) to[open] ++(0,1.1) [ultra thick] [stealth-stealth, scale=1.02] to[bend left] node[above,pos=0.1]{$j1\Omega$} ++(2,0); +\end{circuitikz} +\end{center} + +On demande : +\begin{enumerate} + \item le facteur de dispersion entre les inductances couplées + \item la valeur du courant $\bar{I_a}$ (amplitude et phase) + \item la puissance active, réactive et apparente fournie par la source de tension + \item la valeur de l'impédance $Z$ à placer (voir circuit ci-dessous) afin d'annuler la puissance réactive fournie par la source de tension +\end{enumerate} + +\begin{center} +\begin{circuitikz}[american] + \draw (0,0) to[V, l=$24V\angle 0$] ++(0,2) -- node[flowarrow]{$\Bar{I_a}$} ++(1,0) to[generic=$Z$] ++(2,0) to[R=$1\Omega$] ++(2,0) + coordinate(SPLIT) to[american inductor=$j2\Omega$] node[below](a){$\bullet$} ++(2,0) to[C=$-j2\Omega$] ++(2,0) node[below](b){$\bullet$} + to[american inductor=$j2\Omega$] ++(2,0) to[R=$1\Omega$] ++(0,-2) -- (0,0); + \draw (SPLIT) to[C=$-j1\Omega$,*-*] ++(0,-2); + \draw (a) to[open] ++(0,1.1) [ultra thick] [stealth-stealth, scale=1.02] to[bend left] node[above,pos=0.1]{$j1\Omega$} ++(2,0); +\end{circuitikz} +\end{center} + +\nosolution + +\newpage +\section{Question Craeye : transitoire} + +Soit le circuit suivant, avec l'interrupteur passant de A à B en $t=0$. \\ +Sachant que $\omega = 2\cdot10^6$ $rad/s$, $R = 1$ $k\Omega$, $L = 1$ $mH$ et $C = 1/6$ $nF$, calculer la valeur de $V_o(t)$ pour $t > 0$.\newline +Exprimez le terme harmonique de la réponse sous forme paramétrique.\newline +\textbf{Point bonus :} trouvez la valeur des coefficients du terme harmonique. + +\begin{center} +\begin{circuitikz}[american] + \draw (0,0) to[sV, v^<=$\cos{(\omega t)}$] ++(0,2) to[R=$R$] ++(0,2.5) -- ++(2,0) node[above](B){\textbf{B}} + node[spdt, scale=-1, anchor=out 2](Sw){}; + \draw (0,0) -- (0,0 -| Sw.out 1) coordinate(a) to[V=$2V$] ++(0,2) to[R=$R$] (Sw.out 1) node[left]{\textbf{A}}; + \draw (Sw.in) -- ++(1,0) coordinate(b) to[C, a_=$C$] (b |- a) coordinate(c) -- (a); + \draw (b) to[L=$L$] ++(2,0) coordinate(d) to[R, a_=$R$,v^=$V_o$] (d |- c) -- (c); +\end{circuitikz} +\end{center} + +\nosolution + +\end{document}